LSAT 90 – Section 4 – Question 23

You need a full course to see this video. Enroll now and get started in less than a minute.

Target time: 1:07

This is question data from the 7Sage LSAT Scorer. You can score your LSATs, track your results, and analyze your performance with pretty charts and vital statistics - all with a Free Account ← sign up in less than 10 seconds

Question
QuickView
Type Tags Answer
Choices
Curve Question
Difficulty
Psg/Game/S
Difficulty
Explanation
PT90 S4 Q23
+LR
Flaw or descriptive weakening +Flaw
A
13%
154
B
9%
154
C
1%
149
D
2%
152
E
74%
163
142
151
159
+Medium 148.293 +SubsectionMedium

This is a Flaw/Descriptive Weakening question.

You might be able to figure out the flaw before looking at the answer choices since we're dealing with a repetitive causal logic flaw.

The official says that six months ago, the fines for parking violations on the city’s streets were raised to help pay for the parking garage that had just opened. Since then, parking violations on our streets have dropped by 50 percent. These are our premises, and here comes the conclusion. Hence, if we want there to be even fewer parking violations, the fines should be raised again.

There is an unstated causal assumption. The official makes this recommendation because she assumes that the fines increasing not only coincided with but caused the reduction in parking violations. This could be true, but remember that two things happened: fines were increased and a new parking garage opened. Do we know that it is not the parking garage that caused the reduction? I mean, it's a parking garage. It's for parking cars. I am not saying it has to be, but simply that the official’s reasoning is flawed because she overlooks this. She assumes increased fines —cause→ reduction without considering parking garage —cause→ reduction.

This error can be phrased in a couple of different ways, but let's first look at how it is phrased in the correct answer choice and then play around with alternative phrasings.

Correct Answer Choice (E) says the argument fails to establish that the initial decrease in parking violations was not due to the availability of additional parking spaces (from the newly opened parking garage). This is great. To establish that the argument’s preferred explanation is correct, we have to establish that it is not the alternative explanation that caused the reduction. But like I said, this is not the only way to phrase this flaw. Let’s look at (A).

Answer Choice (A) says the argument takes a possible effect of a reduction to be a possible cause of that reduction. I think one of the reasons why (A) is attractive is that the correct answer could have been phrased similarly to (A). It could have said that the argument takes a possible cause of a reduction to be the definite cause of that reduction. This could have been the correct answer. It would have been using a positive phrasing, "takes a possible cause..." as opposed to the negative phrasing in (E) of “fails to establish...” but that's fine. There are many ways to phrase the error.

But the problem with (A) is that the argument didn't take the effect of the parking violations going down to be the cause of the fines going up. That is, the argument didn't take the loss of revenue from fewer parking violations to be the cause of increased fines.

Answer Choice (B) says the argument takes for granted that raising fines a second time will reduce parking violations at least as much as it did the first time. Takes for granted is also language that the correct answer could have used. The argument "takes for granted that a possible cause of a reduction is the definite cause of that reduction." But, of course, this is not what (B) says. What (B) says is not even descriptively accurate. The official does not say that raising fines will get us another 50 percent reduction. She merely says, “if we want to further reduce,” and does not say by how much.

But (B) is not great even if it passed the descriptive accuracy test. Say we changed (B) and it now reads, “takes for granted that raising fines a second time will reduce parking violations like it did the first time.” This is better since it is a flaw to assume that something will work a second time just because it worked the first time. But (B) still misses the major flaw that it worked the first time.

Answer Choice (C) says the argument fails to take into account the financial benefits the city is now deriving from fines for parking violations. So it did. Okay, (C) is descriptively accurate. But how is this the flaw? Did the official need to talk about these benefits, like how we might use these funds to hire more traffic police or something? This has nothing to do with evaluating the argument.

Answer Choice (D) says the argument takes for granted that people who park their cars illegally would prefer to park their cars legally. Of course people would prefer to park legally. Okay, I know that some people hate being told what to do and believe that it is their right as red-blooded Americans to park on their neighbor's lawn, but the overwhelming majority of people do not.

(D) is indeed an assumption that runs through the whole argument, because whether it is the fines or the new garage that resulted in the reduction, both carry the assumption that people want to park legally. But the best we can do is to say that (D) is descriptively accurate. But it doesn't describe the flaw.

Take PrepTest

Review Results

Leave a Reply